Legislator: The recently passed highway bill is clearly very unpopular with voters. After all, polls predict that the...

EmilyMarieMenendez on August 25, 2018

Why Answer Choice A?

For this passage, not sure how to break it down. Can someone explain the thought process and how A is correct?

Reply
Create a free account to read and take part in forum discussions.

Already have an account? log in

Mehran on August 25, 2018

Let's start, as always, with the stimulus. This one consists of an argument: the legislator's conclusion is "the recently passed highway bill is clearly very unpopular with voters." Why? What premise/support is offered? "Polls predict that the majority party, which supported the bill's passage, will lose more than a dozen seats in the upcoming election."

This is a terrible argument. There might be a hundred different reasons why the majority party is going to lose these seats. Why does the legislator assume that the imminent loss of seats is tied in any way to the recently passed highway bill?

The question stem here is an Errors of Reasoning question that asks you to identify the flaw. Answer choice (A) identifies the error in reasoning demonstrated by the stimulus: the legislator "gives no reason to think that the predicted election outcome would be different if the majority party had not supported the bill." That's right - there is no cause and effect link established here, yet one is definitely assumed by the legislator.

Hope this helps.